Womens’ Health KSA

2019 UAFM CME & SKI

Aaron Starbuck & Marlin Christianson Utah Valley Family Medicine Residency American Board of Family Medicine Knowledge Self‐Assessment Questions: Women's Health

Note: The order in which these questions are listed is the order in which they will be presented the first time through the Knowledge Self‐Assessment. On subsequent visits to the assessment, the questions will be presented in groups organized by competency (content area). 1 . q1 A 3 9 ‐year‐old gra vida 2 para 2 who previously had regular menses presents with an 8 ‐month history of . She jogs a few tim es a week but d o es not engage in more vigorou s a c tivity, and has no recent history of w e ight changes. She has hypothyroidism and takes le vothyroxine (Synthroid) but has n o other medical problems. She is s e xua lly a c tive with her husband and her family history is unremarkable.

On examination her height is 175 c m (68 in) and her BMI is 26.3 kg/m 2 . Her vital signs are normal and she has n o acne or hirsute features. Breast, genital, and pelvic exam inations are a ls o normal. 1.q2 W h ich of the following would be appropriate in the initial evaluation of this patient? (M ark all that are true.)

○ A test ○ FSH and LH leve ls ○ A TSH level ○ A prolactin level ○ A s e rum testosterone le vel ○ Pelvic u ltrason ography 1. a1 Wh ich of the following would be appropriate in the initial evaluation of this patient? (M ark all that are true.)

○ A

Secondary Amenorrhea • cessation of menses: w/ HO regular cycles for 3 mo or irregular for >6 mo  Check for pregnancy 1.a2 Wh ich of the following would be appropriate in the initial evaluation of this patient? (M ark all that are true.)

○ A pregnancy test ○ FSH and LH lev e ls

Primary ovarian insufficiency (premature ovarian failure) • <40 yo oligo- or amenorrhea; elevated FSH & LH x2 >1 mo apart. Usually idiopathic  Check FSH & LH Functional Amenorrhea • Low FSH & LH; poor nutrition; excessive exercise  Check FSH & LH 1. a3 Wh ich of the following would be appropriate in the initial evaluation of this patient? (M ark all that are true.)

○ A pregnancy test

○ FSH and L H lev e ls ○ A TSH lev el ○ A prolactin level

Thyroid Disease • Prolactin abnormalities w/ amenorrhea Check TSH & Prolactin 1. a4 Wh ich of the following would be appropriate in the initial evaluation of this patient? (M ark all that are true.)

○ A pregnancy test

○ FSH and L H lev e ls

○ A TSH lev el

○ A prolactin level ○ A s e rum testosterone le vel

No hyperandrogenisam on PE 1.a5 Wh ich of the following would be appropriate in the initial evaluation of this patient? (M ark all that are true.)

○ A pregnancy test

○ FSH and L H lev e ls

○ A TSH lev el

○ A prolactin level

○ A s e rum testosterone l e vel ○ Pelvic u ltrason ography

H&P no structural abnormality as cause MRI if visual disturbances, or other signs of increased intracranial pressure, and the initial prolactin level were elevated, this would be concerning for neoplasm 2.Q1 A 3 2 ‐year ‐old female comes t o your office because she has been unable to become pregnant after trying to con c e ive for 1 8 months. Her previous medical history also includes mild endometriosis diagnosed by laparoscopy.

W h ich one o f the following treatments for endometriosis improves pregnancy rates?

A) NSAIDs B) Oral con t raceptive pills C) D a n a zol D) Gonadotropin ‐releasing hormone analogues E) Surgical treatme nt of endometriosis lesions 2.A1 A 3 2 ‐year‐old female comes to your office because she has been unable to become pregnant after trying to con c e ive for 1 8 months. Her previous medical history also includes mild endometriosis diagnosed by laparoscopy.

W h ich one o f the following treatments for endometriosis improves pregnancy rates?

Improves Pain A) NSAIDs B) Oral con traceptive pills C) D a n a zol D) Gonadotropin ‐releasing hormone analogues E) Surgical treatme nt of endometriosis lesions 3. Q1 True statements regarding substance abuse in wom en include which o f the following? (Mark all that are true.)

○ Compared with men, women have a quicker progression from first using a substanc e to developing dependence ○ Women are less likely than men to attempt to quit smoking and more likely to relapse if they do attempt t o quit

○ Women are less like ly than men to use marijuana ○ Women are le s s like ly than men to seek treatment for sedative and anxiolytic abuse 3. A1 True statements regarding substance abuse in wom en include which o f the following? (Mark all that are true.)

○ Compared with men, women have a quicker progression from first using a substanc e to developing dependence ○ Women are less likely than men to attempt to quit smoking and more likely to relapse if they do attempt t o quit

○ Women are less like ly than men to use marijuana ○ Women are le s s like ly than men to seek treatment for sedative and anxiolytic abuse 4.Q1 A 44‐year‐old native of M e xico comes to your office to establish care after r e locating to your community. She delivered two term babies weighing 4200 g (9 lb 4 oz) and 4700 g (10 lb 6 oz) 1 6 and 1 4 years ago, r e spective ly. She was dia g n o se d w ith g e statio nal d iab etes mellitus in both pre g n a ncie s. H e r mother and father both h a v e ty pe 2 d iabetes that w a s d iag n o se d in their 50s. O n examination the patient’s BMI is 31.0 kg/m 2 and her bloo d pre ssure is 136/ 82 mm Hg. 4. Q1 Elemen ts of this patient’s history that suggest she should b e sc reen ed for ty p e 2 diabetes mellitus according to the U.S. Preventive Ser v ice s Task Force include which of the following? (Mark all that are true.)

○ H e r BMI ○ H e r b lood p ressure ○ Her ethnicity ○ A first degree relative with type 2 diabetes ○ A history of gestationa l diabetes 4. A1 Elemen ts of this patient’s history that suggest she should b e sc reen ed for ty p e 2 diabetes mellitus according to the U.S. Preventive Ser v ice s Task Force include which of the following? (Mark all that are true.)

○ H e r BMI ○ H e r b lood p ressure ○ Her ethnicity ○ A first degree relative with type 2 diabetes ○ A history of gestationa l diabetes 4. A2 USPSTF

• 40- 70 yo w/ BMI • GDM >25 • Ethnicity • FHX • PCS

ADA • Activity • 1st Degree relative with DM • High Risk Race (AA, Latino, Native American, Asian, Pacific Islander) • Delivery baby >4080 g (9 lbs) • HDL <35 or Triglycerides >250 • PCOS • H/O A1C >5.7%; glucose tolerance, abnormal fasting glucose • Clinical S/Sx of insulin resistance • CVD 5.Q1 A 23‐year‐old gravida 2 para 1 presents to your office following a positive home pregnancy test. Two days ago she started to have some light spotting and cramping. Her last menstrual period was 7 weeks ago. She reports that this is a desired pregnanc y. You perform ultrasonography in the office . 5. Q2 Wh ich of the following would be diagno stic of pregnancy fa ilure? (Mark all that are true.)

○ A crown ‐rump length ≥ 7 mm and no heartbeat ○ A mean s a c diameter ≥ 2 5 mm and no embryo ○ The a b sence o f a n embryo ○ A quantitative hCG level that fails to double in 48 hours ○ A n open c e rvica l os o n speculum examination with products of conception in the vaginal vault 5. A3 • Symptoms of pain or bleeding in early pregnancy • currently viable intrauterine pregnancy, • a failed intrauterine pregnancy, • ectopic pregnancy • Multispecialty Panel 2013 Exclusion Criteria for Viable IUP • Due to consequences to failing to diagnose viable pregnancy criteria more stringent 5. A4 Wh ich of the following would be diagno stic of pregnancy fa ilure? (Mark all that are true.)

○ A crown ‐rump length ≥ 7 mm and no heartbeat ○ A mean s a c diameter ≥ 2 5 mm and no embryo ○ The a b sence o f a n embryo (could be early pregnancy) ○ A quantitative hCG level that fails to double in 48 hours(could be ectopic pregnancy). ○ A n open c e rvica l os o n speculum examination with products of conception in the vaginal vault 6. Q1 Exercise and physical a c t iv ity in women have been associated with improvement in which of the following? (Mark all that are true.)

○ M e n o p a usal hot flashes ○ D e pre s sion ○ Brea st cancer risk ○ Osteoporotic fracture risk 6. Q2 Exercise and physical a c t iv ity in women have been associated with improvement in which of the following? (Mark all that are true.)

○ M e n o p a usal hot flashes 6. Q3 Exercise and physical a c t iv ity in women have been associated with improvement in which of the following? (Mark all that are true.)

○ M e n o p a usal hot flashes ○ D e pre s s ion ○ Brea st cancer risk ○ Osteoporotic fracture risk

Physical activity shown to reduce risk/symptoms of: heart disease stroke diabetes mellitus colon cancer dementia. Fibromyalgia Insomnia chronic back pain chronic fatigue syndrome. 7 . A1 A 3 6 ‐year‐old female presents with knee pain that she has been treating with ibuprofen. She gave birth to her third child 2 months ago and is breastfeeding without problems. She smokes half a pack of cigarettes per day. Her pregnancy was complica ted by preeclampsia but she had normal blood pre ssure before this pregnancy. On examination her blood pre ssure is 170/95 mm Hg. You re fer her to physical therapy for her knee pain and have her return for follow‐up o f her bloo d pre ssure e le v atio n. A t the next visit her bloo d pre ssure is 168/98 mm Hg. She has no symptoms re lated to the b lood pressure elevation. 7. A2 True statements regarding this patient’s hypertension include which of the following? (Mark all that are true.)

○ It may be due to unresolved preeclampsia ○ Her preeclampsia makes her more like ly to develop chron ic hypertension ○ Ibuprofen may be contributing to the bloo d pressure elevation ○ She should be treated with an antihypertensive drug ○ All calcium channel blo cke rs sho u ld b e a v oided while she is breastfeeding ○ All β ‐b lo ckers should b e avoided while she is brea stfeeding 7. A3 True statements regarding this patient’s hypertension include which of the following? (Mark all that are true.)

○ It may be due to unresolved preeclampsia

Preeclampsia • HTN usually resolves by 2 weeks, but may last 12, and rarely 6 mo. 7. A4 True statements regarding this patient’s hypertension include which of the following? (Mark all that are true.)

○ I t may be due to unresolved preeclampsia ○ Her preeclampsia makes her more like ly to develop chron ic hypertension

Preeclampsia • 2x as likely to develop HTN in 12 mo after pregnancy 7. A5 True statements regarding this patient’s hypertension include which of the following? (Mark all that are true.)

○ I t may be due to unresolved preeclampsia

○ Her preeclampsia makes her more like l y to develop chron i c hypertension

○ Ibuprofen may be contributing to the bloo d pressure elevation

NSAIDs could contribute to BP elevation, a NON-NSAID should be used. 7. A6 True statements regarding this patient’s hypertension include which of the following? (Mark all that are true.)

○ I t may be due to unresolved preeclampsia

○ Her preeclampsia makes her more like l y to develop chron i c hypertension

○ Ibuprofen may be contributing to the bloo d pressure elevation

○ She should be treated with an antihypertensive drug ○ All calcium channel blo cke rs sho u ld b e a v oided while she is breastfeeding

Small amounts of CCB are released in breast milk. AAP CCB compatible with breastfeeding, however there may be better options. 7. A7 True statements regarding this patient’s hypertension include which of the following? (Mark all that are true.)

○ I t may be due to unresolved preeclampsia

○ Her preeclampsia makes her more like l y to develop chron i c hypertension

○ Ibuprofen may be contributing to the bloo d pressure elevation

○ She should be treated with an antihypertensive drug

○ All calcium channel blo cke rs sho u ld b e a v oided while she is breastfeeding ○ All β ‐b lo ckers should b e avoided while she is brea stfeeding

Smaller amounts of BBs labetalol, metoprolol and propranolol are released in breast milk than other (and are therefore preferred). 8. Q1 A 67 yo female sees you because of urinary incontinence. She reports episodes of an abrupt need to void, a c c o mpa n ied b y leaka g e o f urine before she has a chance to get t o the bathroom. She does not have leakage o f urine with coughing, sneezing, or laughing. She has mild hypertension controlled with amlo dipine (Norvasc), 5 mg daily. Otherwise she has no medical problems and takes no other medications. 8. Q2 Interventions likely to be beneficial for this patient include which of the following? (Mark all that are true.)

○ α ‐Adrenergic a g o n ists suc h a s pse u doep hedrin e or p henyleph rin e ○ β ‐Adrenergic agonists such a s mirabegron ( Myrbetriq) ○ Pelvic floor m u scle exercises ○ Posterior tibial nerve stimulators ○ Retropubic urethropexy 8. A1 Interventions likely t o be beneficial for this patient include which of the following? (Mark all that are true.)

○ α ‐Adrenergic a g o n ists suc h a s pse u doep hedrin e or p henyleph rin e

Urge Incontinence • α-Adrenergic agonists cause urethral constriction not indicated; only weak evidence to support their superiority over placebo (SOR B) 8. A2 Interventions likely t o be beneficial for this patient include which of the following? (Mark all that are true.)

○ α ‐ Adrenergic a g o n ists suc h a s pse u doep hedri n e or p henyleph r i n e ○ β ‐Adrenergic agonists such a s mirabegron ( Myrbetriq)

Urge Incontinence • Mirabegron, a member of a new class of drugs used to treat urge incontinence, acts on β3-adrenergic receptors to relax the detrusor. 8. A3 Interventions likely t o be beneficial for this patient include which of the following? (Mark all that are true.)

○ α ‐ Adrenergic a g o n ists suc h a s pse u doep hedri n e or p henyleph r i n e

○ β ‐ Adrenergic agonists such a s mirabegron ( Myrbetriq) ○ Pelvic floor m u scle exercises

Urge Incontinence • pelvic floor muscle exercises and bladder training are first-line treatment for urge incontinence. 8. A4 Interventions likely t o be beneficial for this patient include which of the following? (Mark all that are true.)

○ α ‐ Adrenergic a g o n ists suc h a s pse u doep hedri n e or p henyleph r i n e

○ β ‐ Adrenergic agonists such a s mirabegron ( Myrbetriq)

○ Pelvic floor m u scle exercises ○ Posterior tibial nerve stimulators

Urge Incontinence • FDA has approved electrical neuromodulation devices not respondive to behavioral interventions • the posterior tibial nerve, which shares a common nerve root with innervation of the bladder, are the most widely used devices. • works at least as well as medication, reducing urge incontinence in up to 75% of patients (SOR B). 9. Q1 True statements about osteoarthritis in women include which of the following? (Mark all that are true.)

○ Osteoarthritis is more common in women than in men at all ages ○ Women tend to present with osteoarthritis at earlie r stages than men ○ Physicians are more likely t o refer men for joint replacement surgery compared to women with the same degree of symptoms

○ Outcomes from total joint arthroplasty are similar in women and men 9. A1 True statements about osteoarthritis in women include which of the following? (Mark all that are true.)

○ Osteoarthritis is more common in women than in men at all ages

Osteoarthritis • Equal rates until age 45 after that more women 9. A2 True statements about osteoarthritis in women include which of the following? (Mark all that are true.)

○ Osteoarthritis is more common i n women than i n men at all ages ○ Women tend to present with osteoarthritis at earlie r stages than men

Osteoarthritis • Equal rates until age 45 after that more women. • Women typically present with worse symptoms, including greater complaints of pain and disability, and more gait changes. • More advanced radiographic findings are also common among women. 9. A3 True statements about osteoarthritis in women include which of the following? (Mark all that are true.)

○ Osteoarthritis is more common i n women than i n men at all ages

○ Women tend to present with osteoarthritis at earlie r stages than men ○ Physicians are more likely t o refer men for joint replacement surgery compared to women with the same degree of symptoms

Osteoarthritis • Studies have shown that men are much more likely to get a recommendation for knee replacement compared to women with the same presentation. 9. A4 True statements about osteoarthritis in women include which of the following? (Mark all that are true.)

○ Osteoarthritis is more common i n women than i n men at all ages

○ Women tend to present with osteoarthritis at earlie r stages than men ○ Physicians are more likely t o refer men for joint replacement surgery compared to women with the same degree of symptoms

Osteoarthritis • Studies have shown that men are much more likely to get a recommendation for knee replacement compared to women with the same presentation. 9. A5 True statements about osteoarthritis in women include which of the following? (Mark all that are true.)

○ Osteoarthritis is more common i n women than i n men at all ages

○ Women tend to present with osteoarthritis at earlie r stages than men ○ Physicians are more likely t o refer men for joint replacement surgery compared to women with the same degree of symptoms ○ Outcomes from total joint arthroplasty are similar in women and men

Osteoarthritis • benefit from total joint replacement as much as their male counterparts in terms of its impact on pain and functional outcomes 10.Q1 A 4 0 ‐year ‐old female comes to your office for a well wom an examination and asks about getting a mammogram. She has no family history of breast can cer and has no breast complaints. 10. Q2 Appropriate a d vice for this patient includes which o f the following? (Mark all that are true.)

○ Mammo graphy screening for women in their 40s reduces mortality from breast cance r

○ U p to 60% of women in their 40s who have a mammogram will be called back for extra views

○ The American Cancer Society recommends annual mammograms for all women sta r ting at age 40

○ The American College o f Obstetricians and Gynecologists recommends annual mammograms for a ll women sta r ting at age 40

○ The American College of Radiology recommends annual mammograms for all women sta r ting at age 40

○ The U .S . Preventive Services Task Force recommends again s t mammograms for women in their 40s 10. A1 Appropriate a d vice for this patient includes which o f the following? (Mark all that are true.)

○ Mammo graphy screening for women in their 40s reduces mortality from breast cance r

Mammography • 1 life is saved for every 1000 40 yo who get mamo. yearly for 10 yrs 10. A2 Appropriate a d vice for this patient includes which o f the following? (Mark all that are true.)

○ Mammo graphy screening for women i n their 40s reduces mortality from breast cance r ○ U p to 60% of women in their 40s who have a mammogram will be called back for extra views 10. A3 Appropriate a d vice for this patient includes which o f the following? (Mark all that are true.)

○ Mammo graphy screening for women i n their 40s reduces mortality from breast cance r ○ U p to 60% of women i n their 40s who have a mammogram will be called back for extra views

○ The American Cancer Society recommends annual mammograms for all women sta r ting at age 40

ACS Mammography • Start age 45 annual until 54. • Every 2 years after 55 yo. 10. A4 Appropriate a d vice for this patient includes which o f the following? (Mark all that are true.)

○ Mammo graphy screening for women i n their 40s reduces mortality from breast cance r ○ U p to 60% of women i n their 40s who have a mammogram will be called back for extra views ○ The American Cancer Society recommends annual mammograms for all women sta r ting at age 40 ○ The American College o f Obstetricians and Gynecologists recommends annual mammograms for a ll women sta r ting at age 40

○ The American College of Radiology recommends annual mammograms for all women sta r ting at age 40

ACOG & American College of Radiology Mammography ○ Annually starting at 40 yo. 10. A5 Appropriate a d vice for this patient includes which o f the following? (Mark all that are true.)

○ Mammo graphy screening for women i n their 40s reduces mortality from breast cance r ○ U p to 60% of women i n their 40s who have a mammogram will be called back for extra views ○ The American Cancer Society recommends annual mammograms for all women sta r ting at age 40 ○ The American College o f Obstetricians and Gynecologists recommends annual mammograms for a l l women sta r ting at age 40 ○ The American College of Radiology recommends annual mammograms for all women sta r ting at age 40

○ The U .S . Preventive Services Task Force recommends again s t mammograms for women in their 40s (D recommendation) 11. Q1 A 2 6 ‐year‐old female who is 2 months post partum presents to your office with symptoms o f hyperthyroidism. A neck examination is

normal. Labo rato ry testing reveals a low TSH level and elevated free T 4

and free T 3 levels.

W h ich one of the following would be most appropriate?

A) A β ‐blocker B) Ultrasonography of the thyroid C) L e vothyroxine (Synthroid) D) Methimazole (Tapazole) E) Radioactiveiodine 11. A1 W h ich one of the following would be most appropriate?

A) A β ‐ blocker B) Ultrasonography of the thyroid C) L e vothyroxine (Synthroid) D) Methimazole (Tapazole) E) Radioactiveiodine

Postpartum Thyroiditis (occurs within 1 yr of pregnancy) ○ Increase immune function postpartum following immune suppression of pregnancy.

○ High recurring rate in subsequent pregnancy ○ Hyperthyroid 1- 2 mo - >hypo 4- 6 mo - > 25% permanent hypo reast euthyroid

○ A β ‐blocker used to treat symptoms ○ Normal PE no US needed 12. Q1 A 22‐year‐old female comes to your office following an unprotected s exua l encounter 2 nights ago. She does not wish to become pregnant.

Options for in the United States include which of the following? (Mark all that are true.)

(Plan B One ‐Ste p) ○ Ulipristal (Ella) ○ The copper IUD (ParaGard) ○ The levonorgestrel IUD (Mirena) 12. A 22‐year‐old female comes to your office following an unprotected s exua l encounter 2 nights ago. She does not wish to become pregnant.

Options for emergency contraception in the United States include which of the following? (Mark all that are true.)

○ Levonorgestrel (Plan B One ‐Ste p) ○ Ulipristal (Ella) ○ The copper IUD (ParaGard) ○ The levonorgestrel IUD (Mirena) 12.A1 Emergency contraception

○ Levonorgestrel (Plan B One ‐Ste p) ○ Available OTC no age restriction

○ Ulipristal (Ella) ○ Selective receptor approved in 2010 by FDA Rx required

○ The copper IUD (ParaGard) ○ Highly effective if used w/in 5 days

○ The levonorgestrel IUD (Mirena)

○ Not been studied 13.Q1 A t a well woman visit, a 5 2 ‐year‐old female mentions that she is no longer interested in having sex with her husband. They have been married for 2 2 years and her unwillingness to have s e x is leading to arguments. Further questioning indica tes that sex is not painful and lubrication is adequate. A pelvic examination is normal.

You are considering a diagno sis of female s exual interest/arousal disorder. 13. Q2 Wh ich of the following have eviden c e o f effectiveness for tre ating this problem? (Mark a ll that are true.)

○ Bupropion (Wellbutrin) ○ Flib a n se rin ( Addyi) ○ S e rtraline (Zoloft) ○ Sildenafil (Viagra) ○ Testosterone ○ Mindfulness‐b a se d th erapy 13. A2 W h ich of the following have eviden c e o f effectiveness for tre ating female s exual interest/arousal disorder.

○ Bupropion (Wellbutrin) ○ Flib a n se rin (Addyi) ○ S e rtraline (Zoloft) ○ Sildenafil (Viagra) ○ Testosterone ○ Mindfulness‐b a se d th erapy 13. A3 W h ich of the following have eviden c e o f effectiveness for female s exual interest/arousal disorder.

○ Bupropion (Wellbutrin) ○ Flib a n se rin (Addyi)

○ S e rtraline (Zoloft) ○ Sildenafil (Viagra) ○ Testosterone

○ Mindfulness ‐ b a s e d t h erapy

Effective at increasing sexual desire. 13. A2 F emale s exual interest/arousal disorder.

○ Bupropion (Wellbutrin)

○ Fli b a n s e r in (Addyi ) ○ S e rtraline (Zoloft) ○ SSRIs interfere with sexual function

○ Sildenafil (Viagra) ○ May improve arousal, genital stimulation, lubrication and blood flow to vagina and clitoris, but does not improve sexual desire.

○ Testosterone ○ Mindfulness‐b a se d th erapy

○ Increase desire. Sex therapy, CBT, skills, & marital therapy help several areas. 14.Q1 Mutations in BRCA1 o r BRCA2 tumor suppressor genes are associated with which types of cancer? (Mark a ll that are true.)

○ Brea st ○ Lung ○ Ovarian ○ Pancreatic ○ Renal cell 14.Q2 Mutations in BRCA1 o r BRCA2 tumor suppressor genes are associated with which types of cancer? (Mark a ll that are true.)

○ Brea st ○ Lung ○ Ovarian ○ Pancreatic ○ Renal cell CA Risk

o Fe(male) Breast o Fallopian o Prostate o Ovarian o Pancreas o Melanoma o Peritoneal o Colon 15.Q1 A 47‐year‐old gravida 2 para 2 reports regular menses but has noted spotting between periods for 6 months. She has moderate osteoarthritis but has no other health problems. Her only medication is ibuprofen, and she has had a and is there fore not using any form of contraceptive. She has no history o f postpartum hemorrhage, bleeding r e lated to surgery, or easy bruising. On examination she has a BMI o f 25.9 kg/m2 and is normotensive. A pelvic examination is n o rmal. A urine pregnancy test is negative and TSH and prolactin le vels and a hematocrit are n o rmal. 15. Wh ich one o f the following would be most appropriate at this p o int?

A) Reassurance that this is normal B) A levonorgestrel IUD (M iren a) C) Transvaginal ultrasonography to measu re the uterine lining D) A n endometrial b io p sy E) MRI 15. Wh ich one o f the following would be most appropriate at this p o int?

A) Reassurance that this is normal menopause B) A levonorgestrel IUD (M iren a) C) Transvaginal ultrasonography to measu re the uterine lining D) A n endometrial b io p sy E) MRI

Age 45 years to menopause – In women who are ovulatory, any AUB, including . In any woman, bleeding that is frequent (interval between the onset of bleeding episodes is <21 days), heavy, or prolonged (>5 days) 16. A 3 2 ‐year‐old nulligravida see s you to discuss infe rtility. She has no history o f sexua lly transmitted infections and her partner has a son from a previous relationship. She has been having unprotected intercourse with her partner for 1 year. Her cycle lengths vary from 2 8 to 3 3 days. She would like to know when she is m o s t fertile. Appropriate a d vice regarding and ovulation monitoring would include which of the following? (Mark all that are true.)

○ There is strong evidence that continuous monitoring for ovulation increases pregnanc y rate s

○ In each cycle, a woman is most fe rtile for the 5 days leading up to ovulation. ○ Basal body temperature and c e rvica l mucus changes are reliable predictors of ovu lation in a ll wom en

○ Urine should be collected in the morning to monitor LH Critique: Monitoring ovulation and timing intercourse can lead to significant stress. Most of the data regarding ovulation prediction is unusable or the samples are too small to be of clinical value. Due to survival times of sperm and oocytes, the majority of the fertile window is prior to ovulation, extending from about 5 days before ovulation until several hours after ovulation. Cervical mucus changes occur consistently in some women but not in all women. Basal body temperatures can retrospectively confirm ovulation but are not as useful in predicting ovulation for women with irregular cycles. Midday and evening urine samples correlate better with peak LH levels.

Lindsay TJ, Vitrikas KR: Evaluation and treatment of . Am Fam Physician 2015;91(5):308-314. Manders M, McLindon L, Schulze B, et al: Timed intercourse for couples trying to conceive. Cochrane Database Syst Rev 2015;(3):CD011345 1. 17Significant sex differences characterize depression. True statements regarding the differences in .depression in women compared to men include which of the following? (Mark all that are true.)

○ Given identical symptoms or scores on screening tools, physicians are more likely to diagnose depression in women than in men

○ Women have a twofold higher risk of depression than men

○ Women are hospitalized for depression more than men

○ Women have higher rates of comorbid substance abuse than men

○ Depressed women are more likely to attempt suicide than depressed men

Critique: The prevalence, incidence, and morbidity risk associated with depression are greater throughout life for women. The underlying mechanisms of the differences are unclear. Sex steroids undoubtedly play a role in the pathogenesis, as well as inflammation, sociodemographic demands, and genetics. Gender-specific risk factors that disproportionately affect women include gender-based violence, low income and income inequality, low or subordinate social status and rank, and unremitting responsibility for the care of others. Given identical symptoms or scores on screening tools, physicians are more likely to diagnose depression in women than in men. However, men are the principal users of inpatient depression care.

Women have more suicidal thoughts, but women and men have similar rates of attempting suicide. Men are more likely to successfully commit suicide, however. Men have higher rates of substance abuse, aggression, and risk-taking, and may manifest depression differently than women.

Crosby AE, Han B, Ortega LA, et al: Suicidal thoughts and behaviors among adults aged ≥18 years—United States, 2008–2009. MMWR Surveill Summ 2011;60(13):1-22. Jackson CA, Mishra GD: Depression and risk of stroke in midaged women: A prospective longitudinal study. Stroke 2013;44(6):1555-1560. Martin LA, Neighbors HW, Griffith DM: The experience of symptoms of depression in men vs women: Analysis of the National Comorbidity Survey Replication. JAMA Psychiatry 2013;70(10):1100-1106. Nemeth CL, Harrell CS, Beck KD, Neigh GN: Not all depression is created equal: Sex interacts with disease to precipitate depression. Biol Sex Differ 2013;4(1):8. Schuch JJ, Roest AM, Nolen WA, et al: Gender differences in major depressive disorder: Results from the Netherlands study of depression and anxiety. J Affect Disord 2014;156:156-163. Mental health: Gender and women’s mental health. World Health Organization, 2016

Critique: Fluoxetine, taken daily throughout the , is FDA approved to treat premenstrual dysphoric disorder (PMDD) Approximately 60%–70% of women respond to SSRIs. Paroxetine is also FDA approved to treat PMDD, and can be taken either continuously or during the only. It is contraindicated in the first trimester of pregnancy, so women taking this medication for PMDD should use reliable contraception. Oral contraceptive pills containing ethinyl estradiol and drospirenone are FDA approved to treat both physical and emotional symptoms of PMDD. The levonorgestrel-containing IUD may control abnormal bleeding and decrease , but it is not FDA approved to treat PMDD. As a mood stabilizer, lamotrigine may be helpful in treating some women with PMDD and there is ongoing research on this class of medications, but it is not FDA approved to treat PMDD.

Biggs WS, Demuth RH: and premenstrual dysphoric disorder. Am Fam Physician 2011;84(8):918-924. Marjoribanks J, Brown J, O’Brien PM, Wyatt K: Selective serotonin reuptake inhibitors for premenstrual syndrome. Cochrane Database Syst Rev 2013;(6):CD001396. Pearlstein T: Treatment of premenstrual dysphoric disorder: Therapeutic challenges. Expert Rev Clin Pharmacol 2016;9(4):493-496. 19. In the CDC U .S . Medical Eligibility Crite ria for C o ntrac eptiv e Use 2016 guidelines, category 4 indicates conditions for which there is an unacceptable risk to the user if a particular contraceptive method is used. W h ich one of the following conditions is classified as category 4 for combined h o rmo nal contraception?

A) Adequately controlled hypertension B) The use of lamotrigine (Lamictal) in patients with a stable seizure diso rder C) Systemic lupus erythematosus with positive antiphospholipid antibodies D) A family history o f breast cancer Critique: The CDC U.S. Medical Eligibility Criteria for Contraceptive Use 2016 guidelines use four categories for classifying combined hormonal contraceptives:

Category 1: A condition for which there is no restriction for the use of the contraceptive method Category 2: A condition for which the advantages of using the method generally outweigh the theoretical or proven risks Category 3: A condition for which the theoretical or proven risks usually outweigh the advantages of using the method Category 4: A condition that represents an unacceptable health risk if the contraceptive method is used

A family history of breast cancer is a category 1 condition for the use of combined hormonal contraceptives. Adequately controlled hypertension and the use of lamotrigine with a stable seizure disorder are both category 3 conditions, and systemic lupus erythematosus with positive antiphospholipid antibodies is a category 4 condition.

Curtis KM, Tepper NK, Jatlaoui TC, et al: US Medical Eligibility Criteria for Contraceptive Use, 2016. MMWR Recomm Rep 2016;65(3):1-103. Sabers A, Buchholt JM, Uldall P, Hansen EL: Lamotrigine plasma levels reduced by oral contraceptives. Epilepsy Res 2001;47(1-2):151-154. 20. A 2 4 ‐year‐old female would like to learn more about – based methods for preventing pregnancy. She has a regular menstrual cycle that is a lways 30 days in length.

You should instruct her to avo id intercourse o n which of the following days o f her cycle? (Mark a ll that are true.)

○ Day 12 ○ Day 17 ○ Day 20 ○ On days when she has slippery, stretchy, or transparent c e rvical mucus ○ On the fourth day after her basal body temperature rises more than 1 ° F over baseline

Critique: Fertility awareness–based methods of contraception can be broadly classified into four types: (1) calendar-based methods based on cycle length and counting cycle days, (2) cervical mucus–based methods that rely on observing and tracking vulvar discharge, (3) symptom-thermal methods that combine cervical mucus and basal body temperature observations, and (4) symptom-hormonal methods that combine mucus observation with technology to detect urinary hormonal metabolites associated with ovulation and fertility. The fertile window within each menstrual cycle is about 6 days long: the 5 days before ovulation and the day of ovulation. This is because of the limited viability of both the ovum, which must be fertilized within 12–24 hours of release, and sperm, which remain viable for 3–5 days in cervical mucus and the upper genital tract.

Ovulation occurs 14 days before the first day of the next menstrual period. In a woman who has regular 30-day cycles, ovulation can be estimated to occur on day 16. This patient should avoid intercourse on day 12 of her cycle. Because the ovum can stay alive for up to 24 hours after release, it is also important to avoid intercourse on the day after ovulation (day 17). Because the viability of an ovum is limited to 24 hours, it is not necessary to avoid intercourse on day 20 of her cycle. As a woman’s level rises during the first part of her menstrual cycle it stimulates glands within the to produce slippery, stretchy, and/or transparent mucus that allows for enhanced sperm motility and survival. The final day of this estrogenic cervical mucus coincides with ovulation, so intercourse should be avoided on days when slippery, stretchy, or transparent cervical mucus is noted.

Basal body temperature is the temperature taken upon awakening, before any activity. It must be measured with a thermometer that has fine gradations. The rise in serum progesterone that occurs after ovulation causes a small increase (usually around 1°F) over the baseline temperature. After the third consecutive day of temperature elevation above baseline it is no longer necessary to avoid intercourse.

The failure rate for fertility-awareness methods with typical use is 24%. Because of this, women may want to obtain additional education about using these methods.

Grimes DA, Gallo MF, Grigorieva V, et al: Fertility awareness–based methods for contraception: Systematic review of randomized controlled trials. Contraception 2005;72(2):85-90. Manhart MD, Duane M, Lind A, et al: Fertility awareness–based methods of family planning: A review of effectiveness for avoiding pregnancy using SORT. Osteopathic Family Physician 2013;5(1):2-8. FACTS about Fertility. Fertility Appreciation Collaborative to Teach the Science, 2016

Critique: The majority of screening recommendations are the same for women who have sex with women (WSW) as for other women. WSW-identified women should be screened for cervical cancer according to the same guidelines as women who have sex with men, beginning at age 21. The majority of self-identified WSW indicate that they have had sex with men at some point in their life. Antibodies to HPV 16 and HPV 18, as well as Papanicolaou testing showing high-grade squamous intraepithelial lesions and low-grade squamous intraepithelial lesions, have been reported among WSW who deny ever having male partners.

HPV can be transmitted through skin-to-skin contact and can be sexually transmitted between women. HPV is prevalent in WSW, just as it is in women who have sex with men. Guidelines for HPV vaccination in WSW are the same as in other women. The CDC recommends HPV vaccine for females 11 and 12 years of age (although it can be given as young as age 9), as well as for females 13–26 years of age who have not yet received all doses or completed the vaccine series. The American Society for Colposcopy and Cervical Pathology does not recommend HPV testing in patients under 30 years of age.

Rates of Chlamydia infection are likely higher than previously thought in the WSW population and female-to-female transmission of syphilis and gonorrhea does occur. Therefore, a patient who self-identifies as WSW should have screening for sexually transmitted infections similar to that of other women. Screening for Chlamydia and gonorrhea is recommended for all women under age 25.

Depression screening is recommended for this patient, as rates of suicidal ideation and attempts are higher in WSW compared to heterosexual women; this is especially true during adolescence and in WSW who have been the target of prejudice or discrimination.

Although there is a high frequency of bacterial vaginosis in WSW populations, neither routine screening nor partner treatment is recommended

Saslow D, Solomon D, Lawson HW, et al: American Cancer Society, American Society for Colposcopy and Cervical Pathology, and American Society for Clinical Pathology screening guidelines for the prevention and early detection of cervical cancer. J Low Genit Tract Dis 2012;16(3):175-204. Adelson SL: Practice parameter on gay, lesbian, or bisexual sexual orientation, gender nonconformity, and gender discordance in children and adolescents. J Am Acad Child Adolesc Psychiatry 2012;51(9):957-974. 2015 Sexually transmitted diseases treatment guidelines: Special populations. Centers for Disease Control and Prevention, 2015.

22. A 51‐year‐old premenopausal female sees you to discuss chemoprevention after having a breast bio psy that showed atypical ductal hyperplasia. She is still having periods and her family history includes invasive breast can cer in a sister at age48. Afte r a conversation about risk s and benefits of chemoprevention and possible side effects she decides to start medication to reduce her risk.

W h ich one o f the following would you recommend for this patient?

A) Combined ora l contraceptives B) Letrozole ( Femara) C) Med rox y p rogesterone (Provera ) D) Raloxifene ( Evista) E) Tamoxifen( Soltamox) Critique: Tamoxifen, a systemic estrogen reuptake inhibitor, was FDA approved in 1998 for primary prevention of breast cancer in high-risk women. It can decrease the risk of developing breast cancer (specifically estrogen-receptor– positive breast cancer) by up to 48%. It is the only FDA-approved medication for the chemoprevention of breast cancer in premenopausal women (SOR A). Raloxifene is another selective estrogen receptor modulator like tamoxifen and is approved for the chemoprevention of breast cancer in postmenopausal women, but not premenopausal women (SOR A).

Letrozole is an aromatase inhibitor. Aromatase inhibitors are approved for chemoprevention of breast cancer in postmenopausal women but are not approved for premenopausal women (SOR A). Aromatase inhibitors block the conversion of androgens to estrogen but cannot block ovarian production of estrogen, so they do not work in premenopausal women unless the woman is also taking a gonadotropin-releasing hormone inhibitor.

Combined oral contraceptives can be used for the prevention of ovarian cancer but do not prevent breast cancer (SOR C). Progesterone does not reduce the risk for breast cancer (SOR A).Final Recommendation Statement: Breast Cancer: Medications for Risk Reduction. US Preventive Services Task Force, 2013. Visvanathan K, Hurley P, Bantug E, et al: Use of pharmacologic interventions for breast cancer risk reduction: American Society of Clinical Oncology clinical practice guideline. J Clin Oncol 2013;31(23):2942-2962. Cuzick J, Sestak I, Cawthorn S, et al: Tamoxifen for prevention of breast cancer: Extended long-term follow-up of the IBIS-I breast cancer prevention trial. Lancet Oncol 2015;16(1):67-75. 23. A 14‐year‐old competit ive gymna st presents for a w e ll c h ild examination. Aware o f the hazards of intense athletic training, the patient's mother is concerned that her daughter has n o t yet started her period, whereas m o s t o f her peers have started theirs. O v er the la st year the patient has noted breast development and growth of hair in the pubic and a xilla r y are as, and she has had a growth spurt.

Appropriate advice includes which of the following? (Mark a ll that are true.)

○ The patient should have a laboratory evaluation at this visit ○ The patient’s pubertal development is within the normal range for girls ○ Low body fat is a ssociated with a delay in ○ Increased ske letal growth generally follows menarche in pubertal development

Critique: Puberty is the process leading to physical and sexual maturation that involves the development of secondary sex characteristics, as well as growth, changes in body composition, and psychosocial maturation. The normal age range for the onset of puberty in girls is 8–14 years. The average age at menarche is 12.5 years. This patient has begun puberty, as evidenced by her increased skeletal growth, breast development, and the appearance of pubic and axillary hair. Menarche generally follows peak skeletal growth by about a year.

Girls with low body fat, such as competitive athletes, may have a delay of up to a year or more in menarche. These patients require both reassurance and continued observation. In addition, a detailed history should be taken to investigate dietary intake and exercise patterns to optimize their health.

Primary amenorrhea is defined as failure to start menstruating by age 13 with no secondary sex characteristics, by age 15 with secondary sex characteristics, or within 5 years of breast development.

Practice Committee of the American Society for Reproductive Medicine: Current evaluation of amenorrhea. Fertil Steril 2008;90(5 Suppl):S219-S225. Latronico AC, Brito VN, Carel JC: Causes, diagnosis, and treatment of central precocious puberty. Lancet Diabetes Endocrinol 2016;4(3):265-274. 24 1. True statements about work-life balance include which of the following? (Mark all that are true.)

○ Women and men now spend an equal amount of time on household tasks

○ Female nurses who work rotating night shifts have a higher incidence of heart disease ○ Women who return to work sooner than 3 months post partum have a reduced duration of breastfeeding ○ Perceived control over their work schedule is associated with improved self-reported mental health in women

○ Female primary care physicians are more likely to experience burnout than their male colleagues Critique: Although the time men spend on household tasks is increasing, women continue to spend more time than men on these tasks. Among female registered nurses, a longer duration of rotating night shifts was associated with a statistically significant increase in the risk for coronary heart disease. Returning to work full time before 3 months post partum may reduce a woman’s ability to breastfeed for at least 3 months. Employees who perceive they have more control over their work schedules self-report an improvement in mental health. Several studies have shown higher rates of burnout in female primary care physicians than in their male colleagues, and the likelihood increases with hours worked per week over 40. The median salary for female physicians in one study was also $22,000 less than men when controlling for multiple factors.

US Department of Labor Bureau of Labor and Statistics: American time use survey—2015 results. Vetter C, Devore EE, Wegrzyn LR, et al: Association between rotating night shift work and risk of coronary heart disease among women. JAMA 2016;315(16):1726-1734. Gregory A, Milner S: Editorial: Work-life balance: A matter of choice? Gend Work Organ 2009;16(1):1-13. Shanafelt TD, Boone S, Tan L, et al: Burnout and satisfaction with work-life balance among US physicians relative to the general US population. Arch Intern Med 2012;172(18):1377-1385. Mirkovic KR, Perrine CG, Scanlon KS, Grummer-Strawn LM: Maternity leave duration and full-time/part-time work status are associated with US mothers’ ability to meet breastfeeding intentions. J Hum Lact 2014;30(4):416-419.

1. A 22-year-old female who is currently taking daily oral contraceptive pills would like to switch to a longer- acting contraceptive method. In order to avoid a gap in contraception, which of the following should you recommend? (Mark all that are true.)

○ If switching to medroxyprogesterone acetate (Depo-Provera), take the first shot 7 days before stopping the pill

○ If switching to the /ethinyl estradiol vaginal ring (NuvaRing), insert the ring the day after stopping the pill

○ If switching to the norelgestromin/ethinyl estradiol transdermal patch (Ortho Evra), start the patch the day after stopping the pill

○ If switching to the etonogestrel implant (Nexplanon) or the levonorgestrel IUD (Mirena), the device should be inserted on the day the pill is stopped

○ If switching to the copper IUD (ParaGard), the IUD can be inserted up to 5 days after stopping the pill Critique: Unintended pregnancy can occur during a gap in contraception. The medroxyprogesterone acetate shot reaches full efficacy 7 days after initiation. When switching to this contraceptive method women should continue using the pill for 7 days. Hormones in the etonogestrel/ethinyl estradiol vaginal ring are rapidly absorbed. Therefore, women switching from a pill to the ring should start the ring the day after they take the last pill. The copper IUD becomes effective immediately upon insertion. In addition, it prevents pregnancy via spermicidal mechanisms up to 5 days after unprotected sexual intercourse, so it can be inserted anytime within 5 days of stopping the pill.

Hormone levels take 48 hours to reach a plateau after a woman applies her first norelgestromin/ethinyl estradiol transdermal patch. When women switch from a pill to a patch, a 2-day overlap prevents a decline in hormone levels, so women should start the patch the day before they take the last pill. Etonogestrel, the hormone in the subdermal implant, reaches peak serum levels in 4 days. Women who switch to the implant should continue using the pill for 4 days after insertion of the implant. The levonorgestrel IUD takes 7 days to produce peak hormone concentrations. When switching to this IUD, women should continue taking the pill for 7 days.

Lesnewski R, Prine L, Ginzburg R: Preventing gaps when switching contraceptives. Am Fam Physician 2011;83(5):567-570. How to switch methods. Reproductive Health Access Project, 2015.

1. A 24-year-old female presents with a 2-day history of urinary frequency, urinary hesitancy, and pain with urination. She has no fever, flank pain, or vomiting. She has been treated twice in the past 6 months for urinary tract infections (UTIs) with two 3-day courses of antibiotics. She has had one urine culture positive for Escherichia coli.

True statements regarding this problem include which of the following? (Mark all that are true.)

○ E. coli is the most likely cause of this patient’s infection

○ Cranberry products should be recommended to help prevent symptomatic UTIs ○ With most antibiotic treatments, a 5-day course would be more effective than a 3-day course ○ If the patient’s UTIs are related to sexual intercourse, postcoital antibiotic prophylaxis would be as effective as daily prophylaxis

○ Prophylaxis for recurrent UTIs should be started immediately after the patient completes treatment for this infection

○ This patient should have renal and bladder imaging Critique: Recurrent urinary tract infection (UTI) is defined as two or more UTIs in 6 months or three or more UTIs in 12 months. One study showed that women who diagnosed themselves with UTIs had an 85% positive urine culture rate, which is more accurate than urine dipstick testing (SOR A). Patients should have a positive urine culture to ensure that a UTI is causing symptoms. If a patient has had a positive urine culture, the bacteria identified on culture is the most likely cause of infection. Escherichia coli is responsible for 80% of urinary tract infections in women (SOR A).

Women with recurrent UTIs should be offered daily prophylactic therapy, prophylactic postcoital antibiotics, or patient- initiated treatment. One study showed that postcoital ciprofloxacin was as effective as daily ciprofloxacin, with fewer side effects. Postcoital prophylaxis should be offered to women with UTIs related to sexual intercourse. Another option is patient-initiated treatment (SOR A). Prophylaxis should not be started until a negative culture 1–2 weeks following treatment confirms that the infection has been eradicated (SOR C). Imaging is indicated only for persistent hematuria or bacteriuria involving any bacteria other than E. coli (SOR B).

Three-day courses of treatment for uncomplicated UTIs have been found to be as effective as five-day courses, except with nitrofurantoin (SOR A).

A Cochrane review update in 2012 could find no significant evidence that cranberry products prevent symptomatic UTIs. This review added 14 additional studies to those already reviewed. In their previous reviews on this subject there was some evidence that women with recurrent UTIs who drank cranberry juice regularly had decreased symptomatic UTIs.

Gupta K, Hooton TM, Roberts PL, Stamm WE: Short-course nitrofurantoin for the treatment of acute uncomplicated cystitis in women. Arch Intern Med 2007;167(20):2207-2212. Epp A, Larochelle A, Lovatsis D, et al: Recurrent urinary tract infection. J Obstet Gynaecol Can 2010;32(11):1082-1101. Jepson RG, Williams G, Craig JC: Cranberries for preventing urinary tract infections. Cochrane Database Syst Rev 2012;10:CD001321. Arnold JJ, Hehn LE, Klein DA: Common questions about recurrent urinary tract infections in women. Am Fam Physician 2016;93(7):560-569. 1. A 16-year-old female brings her infant in for well care at 2 weeks of age. During the visit she asks about her 2 contraception options. Her BMI is 34.0 kg/m , she is not breastfeeding, and she has no medical problems.

Which one of the following contraceptive options would be safest and most effective for this patient?

A) A progestin-only pill

B) A combined estrogen/progestin pill

C) The etonogestrel/ethinyl estradiol vaginal ring (NuvaRing)

D) The norelgestromin/ethinyl estradiol patch (Ortho Evra)

E) The etonogestrel implant (Nexplanon)

Critique: The etonogestrel implant has the highest efficacy of all the options listed, is safe in adolescents, and is safe for insertion in nonbreastfeeding women less than 21 days post partum (U.S. Medical Eligibility Criteria for Contraception Use category 1). Furthermore, its efficacy would not be reduced by this patient’s obesity (SOR A). Research subsequent to the FDA approval of the etonogestrel implant has shown that its efficacy is equal to that of the copper IUD across all BMIs, including in women who are overweight and obese. Women who are overweight or obese are at increased risk for venous thromboembolic (VTE) disease, and their risk increases with age. Therefore, non–estrogen-containing, highly effective birth control options are desirable to decrease VTE risk.

An IUD would also be a good choice for this patient. She could use either a copper IUD or a levonorgestrel IUD. There is an increased expulsion rate if it is inserted 2 weeks post partum as opposed to immediately after delivery of the placenta, or after 4 weeks post partum.

Because of the risk of VTE, being less than 21 days post partum in a woman who is not breastfeeding is a category 4 condition (absolute contraindication) for the combination pill, etonogestrel/ethinyl estradiol vaginal ring, and norelgestromin/ethinyl estradiol patch (SOR C). In addition, the combined estrogen/progestin pill is less effective with real- world use than the etonogestrel implant. The progestin-only pill can be safely initiated less than 21 days post partum, but it is less effective than the etonogestrel implant.

Xu H, Wade JA, Peipert JF, et al: Contraceptive failure rates of etonogestrel subdermal implants in overweight and obese women. Obstet Gynecol 2012;120(1):21-26. Klein DA, Arnold JJ, Reese ES: Provision of contraception: Key recommendations from the CDC. Am Fam Physician 2015;91(9):625-633.

1. A 41-year-old female with generalized pain meets the clinical criteria for fibromyalgia. She frequently misses work because of the pain. Her only medication is over-the-counter ibuprofen. Laboratory findings are normal.

Which of the following have been shown to be effective for this problem? (Mark all that are true.)

○ Aerobic exercise

○ Amitriptyline

○ Clonazepam (Klonopin)

○ Cyclobenzaprine

○ Duloxetine (Cymbalta)

○ Gabapentin (Neurontin)

Critique: Because of the wide range of manifestations of thyroid disease and its high frequency in women with menstrual disorders, a TSH level should be checked in women with possible polycystic ovary syndrome (PCOS) (SOR C). A prolactin level should be obtained to rule out hyperprolactinemia as a cause of in women with suspected PCOS (SOR C). Nonclassical congenital adrenal hyperplasia, often referred to as late-onset congenital adrenal hyperplasia, is present in 1%–8% of hirsute women and should be screened for by obtaining a morning 17-hydroxyprogesterone (17-OHP) level (SOR C).

An androgen-secreting tumor is characterized by a rapid onset of virilization symptoms including changes in voice, male pattern androgenic balding, and clitoromegaly. Testing is not indicated in the absence of these symptoms. Primary ovarian insufficiency involves amenorrhea (as opposed to ) combined with symptoms of estrogen deficiency including hot flashes or urogenital symptoms. Testing is not indicated in the absence of these symptoms.

Because Cushing syndrome is extremely rare (1 in 1,000,000 individuals) and screening tests are not 100% sensitive or specific, routine screening for Cushing syndrome in all women with hyperandrogenic chronic anovulation is not indicated. Those who have coexisting signs of Cushing syndrome, including moon facies, a buffalo hump, abdominal striae, centripetal fat distribution, or hypertension, should be screened.Kodner C: Common questions about the diagnosis and management of fibromyalgia. Am Fam Physician 2015;91(7):472-478. Wolfe F, Clauw DJ, Fitzcharles MA, et al: Fibromyalgia criteria and severity scales for clinical and epidemiological studies: A modification of the ACR preliminary diagnostic criteria for fibromyalgia. J Rheumatol 2011;38(6):1113-1122. 1. A 26-year-old female comes to your office with a 2-year history of infertility and confirmed anovulation. The initial evaluation should include which of the following? (Mark all that are true.)

○ A history and physical examination

○ A TSH level

○ A prolactin level

○ Pelvic ultrasonography

○ Karyotype analysis Critique: Primary ovulatory insufficiency (ovulatory dysfunction) is the most commonly identified cause of infertility in women. A history and physical examination can identify menstrual disorders, anatomic abnormalities, signs of androgen excess, and other problems that may guide the subsequent evaluation (SOR C). Hypothyroidism and hyperprolactinemia are relatively common causes of anovulation and require specific treatment (SOR C). Timed FSH and estradiol levels will distinguish between hypothalamic/pituitary, ovarian, and other causes of ovulatory dysfunction (SOR C). Pelvic ultrasonography can identify follicle development but due to cost and logistics it is not recommended as part of the initial evaluation (SOR C). Karyotype analysis may be indicated for certain women, such as those with clinical signs of Turner’s syndrome, but it is not recommended for all women with anovulation (SOR C).

Practice committee of the American Society for Reproductive Medicine: Diagnostic evaluation of the infertile female: A committee opinion. Fertil Steril 2012;98(2):302-307.

1. Consistent use of male is associated with which of the following in women? (Mark all that are true.)

○ An annual of 2% with typical use

○ Decreased acquisition of herpes simplex virus type 2

○ Decreased acquisition of Chlamydia and gonorrhea infections

○ Decreased acquisition of HIV infection

○ Increased clearance of genital HPV infection

○ An increased risk of bacterial vaginosis Critique: Male condoms reduce the risk of many sexually transmitted infections (STIs), including gonorrhea, Chlamydia, herpes, HIV, trichomoniasis, and syphilis. Male use has also been associated with regression of HPV-related abnormalities in men and women and a decreased risk of bacterial vaginosis. With perfect use male condoms have an annual failure rate of 2%. However, the annual pregnancy rate is 18% in actual use. Female condoms have not been as well studied but they do reduce STI risk. The pregnancy rate is slightly higher than with male condoms (21% annual failure rate with actual use and 5% annual failure rate with perfect use).

Holmes KK, Levine R, Weaver M. Effectiveness of condoms in preventing sexually transmitted infections. Bull World Health Organ 2004;82(6):454-461. Condoms and STDs: Fact sheet for public health personnel. Centers for Disease Control and Prevention. 31.Q 1 A 3 5 ‐year‐old female presents to your office for treatment of insomnia. She d isclo ses that she w a s sexually a ssau lted 6 w e e k s ago. She h a s not so ught medical, legal, or psychological counseling sinc e the assa ult. 31. Q2 Appropriate elements o f this patient’s c a re include which of the following? (Mark all that are true.)

○ Te sting for sexually transmitted infe ctions ○ A pregnancy test ○ Evaluation for symptoms of posttraumatic stress disorder, depression, and anxiety

○ A n examination to collect evidence (e.g., a “rape kit” examination) 31. A1 Appropriate elements o f this patient’s c a re include which of the following? (Mark all that are true.)

○ T e sting for sexually transmitted infe ctions ○ A pregnancy test ○ Evaluation for symptoms of posttraumatic stress disorder, depression, and anxiety

○ A n examination to collect evidence (e.g., a “rape kit” examination) 31. A2 Appropriate elements o f this patient’s c a re include which of the following? (Mark all that are true.)

○ T e sting for sexually transmitted infe ctions

○ GC, Chlamydia, syphilis, HIV, Trich, HIV (<72 hrs prophylaxis)

○ A pregnancy test ○ 5% testing and emergency contraception offered if <5 days. 31. A4 Sexual Assault ○ T e sting for sexually transmitted infe ctions

○ A pregnancy test ○ Evaluation for symptoms of posttraumatic stress disorder, depression, and anxiety ○ Sequelae may be severe treating ASAP important to reduce duration and severity of distress. Refer to survivor advocacy organization or MH w/ experience.

○ A n examination to collect evidence (e.g., a “rape kit” examination)

○ Mandatory reporting laws or patients decide.

○ >48- 72 hrs/1 wk for some DNA evidence.

○ Most offices don’t have ability to do. 32. Q1 A 23‐year‐old female presents to your office because of concerns about irregular menstrual periods, which she s ays occur every 2 – 3 months. On examination she has a b lood pressure of 138/82 mm Hg, a pulse rate of 6 6 beats/min, and a BMI of 32.0 kg/m 2 . She has a moderate amount o f coarse, dark hair o n her upper lip, c h in, abdomen, and upper and inner thighs, and she has moderate inflammatory acne o n her face and upper back. The remainder of the examination is unremarkable. 32.A2 Conditions that should be excluded before making a diagnosis of polycystic ovary syndrome in this patient include which of the following? (Mark all that are true.)

○ A n androgen ‐s e creting tumor ○ Cushing sy ndro m e ○ Nonclassical congenital adrenal h y perpla sia ○ Primary ovarian insufficiency ○ Prolactin e xcess ○ Thyroid disease 32.A2 Conditions that should be excluded before making a diagnosis of polycystic ovary syndrome in this patient include which of the following? (Mark all that are true.)

○ A n androgen ‐s e creting tumor ○ Rare 1:1 mil; ○ other symptoms than hyperandrogenic chronic anovulation (moon facies, buffalo hump, HTN, abdominal striae, centripetal fat distribution)

○ Cushing sy ndro m e ○ Rapid virilizaition (voice, male pattern balding, clitoromegaly) 32.A3 Exclude before Dx of PCOS

○ A n androgen ‐s e creting tumor ○ Rapid virilizaition (voice, male pattern balding, clitoromegaly)

○ Cushing sy ndro m e ○ Rare 1:1 mil; ○ other symptoms than hyperandrogenic chronic anovulation (moon facies, buffalo hump, HTN, abdominal striae, centripetal fat distribution)

○ Nonclassical congenital adrenal h y perpla sia

○ Primary ovarian insufficiency ○ Amenorrhea (not oligo). ○ Also estrogen deficiency (hot flashes/urogenital symptoms)

○ Thyroid disease 32.A4 Exclude before Dx of PCOS

○ A n androgen ‐ s e creting tumor ○ Cushing s y ndr o m e ○ Nonclassical congenital adrenal h y perpla sia ○ 1 - 8% of hirsute women get AM 17- OHP

○ Primary ovarian insufficiency ○ Prolactin e x cess ○ Prolactin level to rule out hyperprolactinemia ○ Thyroid disease ○ Common in women with menstrual disorders, check a TSH 33.Q1 A 29‐year‐old female comes to your office after disco v ering a mass in her le ft breast 2 weeks a g o . She says it d o es n o t hurt and she has no other breast compla ints. On examination you detect a 3 ‐c m smooth mass in the upper outer quadrant of her left breast. The re s t of the examination, including examination o f the right breast, is normal. 33. Q2 W h ich one o f the following is the most appropriate next step in this patient’s evaluation?

A) Observation o n ly and follow‐up in 2 months B) Refe rral for diagnostic mammography C) Refe rral for ultrasonography of the breast D)MRI of the breast 33. Q2 W h ich one o f the following is the most appropriate next step in this patient’s evaluation?

A) Observation o n l y and follow‐ up i n 2 months

B) Refe rral for diagnostic mammography C)Refe rral for ultrasonography of the breast

D) MRI of the breast

Breast Mass-

• dominant mass suspicion for CA (10% with increased age and other factors).

• Low risk based on age for CA but not low enough for watchful waiting.

• <30 US; >40 diagnostic mammo; 30- 40 start with either. 34. Q1 A 2 8 ‐year‐old gravida 1 para 1 presents with a 6 ‐month histo ry of bila teral spontaneous nip p le discharge. Her pregnancy was 4 years ago. She has not had any breast pain and there is no history o f trauma. The re view o f systems is negative exc e pt for a 1 ‐year history of oligomenorrhea. Her past medical history is significant for low back pain and bipolar diso r d e r. Her current medications include divalpro ex sodium (Depakote) and respe r idone (Risperdal). 34. Q2 On examination the patient’s vital signs are n o rmal. A thin, milky disc harge is expressed from b o th nipples. The breast examination is otherwise n o rmal, with no masses or adenopathy. The neurologic examination is also normal, including visual field testing. Guaiac testing of the nipple disc harge is negative. Her serum prolactin level is 100 µg/L (N 4 – 23). 34. A1 Appropriate steps at this point include which of the following? (Mark all that are true.)

○ A pregnancy test ○ Thyroid func tio n tests ○ Cytology of the nipple discharge ○ Renal func tio n tests ○ Mammo graphy ○ Brain MRI 34. A2 Bilateral Galactorrhea w/ Hyperprolactinemia

○ A pregnancy test ○ Thyroid func tio n tests (hypothyroidism)

○ Cytology of the nipple discharge ○ Renal func tio n tests

○ Mammo graphy

○ Brai n MRI

• Nipple Stimulation galactorrhea • Renal Insufficiency • No MRI unless cause of prolactin uncertain • No cytology unless pathological nipple discharge • Medications 35. Q1 A 38‐year‐old gravida 3 para 2 who had two vaginal deliveries and a miscarriage sees you to discuss contraception. She is divo rc ed but has re cently become s e xua lly a c tiv e with a new partner. She has hypothyroidism, which is controlled with levothyroxine (Synthroid). She drinks 4 glasses o f wine per week and smokes 1 pack o f cigarettes per day. On examination today her vital signs are n o rmal. 35 A1 Appropriate con tra ceptive options for this patient include which of the following? (Mark all that are true.)

○ A pro g estin‐only pill ○ A combined estrogen/progestin pill ○ A n etonogestrel implant (Nexplanon) ○ A nore lgestromin/ethinyl estradiol patch (Ortho Evra) ○ A copper IUD (ParaGard) ○ A levonorgestrel IUD (M iren a) 35 A2 con tra ceptive options in 38 yo w/ smoking

○ A pro g estin‐only pill

○ A combined estrogen/progestin pill ○ A n etonogestrel implant (Nexplanon) ○ A nore lgestromin/ethinyl estradiol patch (Ortho Evra) ○ A copper IUD (ParaGard) ○ A levonorgestrel IUD (M iren a)

• >35 yo; >15 cig/day CAT 1 (no restrictions) progesterone • >35 yo; >15 cig/day CAT 4 (unacceptable risks) estrogen contraceptives. 36. Q1 A healthy 4 4 ‐year‐old gravida 2 para 2 sees you because of heavy, painful periods that have increased over the past y e a r. She says she has felt weak and has fatigued easily for the past seve ral months. Her menstrual flow often prevents her from p a rticipating in her usual a c tivities . She does not desire a future pregnancy. A physical examination and ultrasonography reveal two la rge intramural uterine le iomyomas. 36. Uterine Fibroids

○ Watchful w a iting

○ Combination ora l contraceptive pills

○ A levonorgestrel‐releasing IUD (Mirena)

○ Uterine artery embolization (avoid hysterectomy, doesn’t optimize fertility, shrinkage 30- 46%; less pain; quicker return to work; more complications and FU; fails with larger fibroids)

○ Abdominal myomectomy(preserve fertility recurrence 15- 30% five years)

○ Hysterectomy (severe symptoms no childbearing desired) 37. A 47‐year‐old female comes to your office to establish care and for follow‐up after having a laparoscopic sleeve gastrectomy 2 months ago. She s ays she has lost about 4 0 pounds and fee ls great. She would like to discuss postsurgical nutrition recommendations. Appropriate advice would include which o f the following? (Mark a ll that are true.)

○ She should have regular monitoring for nutritional status and deficiencie s ○ F luid intake befo re meals enha nce s satiety ○ Food p referen ces tend to b e stable befo re and after surgery ○ Protein intake should be prioritized to ensure at least 80– 9 0 g/day ○ Vitamin B12 suppleme ntation is recommended ○ Iron deficiency is common after b a riatric surgery 38.True statements regarding medroxyprogesterone acetate (Depo ‐ Provera) include which of the following? (Mark all that are true.)

○ It causes permanent b o ne m ineral density loss ○ There is little evidence that it causes more weight gain than other hormonal methods ○ It is not recommended for patients with a h istory o f ven o us thromboembolism ○ Antico n v ulsants can decrease its efficacy 39.Relative energy deficiency in sport (RED‐S), formerly k n own as the female athlete triad, can lead to which of the following? (Mark all that are true.)

○ Card io va sc ular d isea se ○ D e pre ssion ○ Impaired immunity ○ Menstrual dysfunction ○ Osteoporosis 40. A 42‐year‐old female has e xperienc ed progressively worsening pelvic pain for the past 1 0 years. She has dysmenorrhea, dyspareunia, and intermittent pain with b o wel movements. A pelvic examination, testing for sexually transmitted infe ctions, and pelvic ultraso nography are negative. True statements regarding this problem include which o f the following? (Mark all that are true.)

○ A mental health history should be o b tained ○ Physical examination maneuvers can be used to distinguish visceral from myofascial pain ○ SSRIs may provide pain relie f for this patient even if she is n ot depressed ○ Opioid medications are considered first‐lin e treatment for chron ic pelvic pain ○ Laparoscopy is indica ted for all women with chronic pelvic pain 41. A 7 0 ‐year‐old female with well controlled hypertension comes to your office for follow‐up. She has no history o f g a strointestina l bleeding and has not had any falls. Her medications include lisinopril/hydrochlorothiazide ( Zestoretic) and acetaminophen. She smokes half a pack o f cigarettes daily and has for 4 5 years. She has a

BMI o f 38.0 kg/m 2 and her EKG is normal. Pharmacologic option s for stroke prevention recommended by the American Heart Association/American Stroke Association for this patient include which o f the following? (Mark all that are true.)

○ A p ix a b a n (Eliquis) ○ Aspirin ○ C lo p id ogrel (Plavix) ○ D a b ig a tra n (Pra daxa ) 42.A 6 5 ‐year‐old female sees you for follow ‐up to d isc uss the resu lts of her scree n ing DXA sc an. She is a n o nsmo ker with n o risk fac tors for osteoporosis. O n examination today her heig h t is 165 cm (65 in), she

weighs 7 9 kg (174 lb), and her BMI is 29.0 kg/m 2 . Her mean femoral neck density is 0.722 g/cm2 , which correlates w ith a T ‐sc ore o f –2.2, and a FRAX score of 12% for major o steop orotic fracture and 2.4% for h ip fracture. Measures to reduce this patient’s risk of osteoporotic fracture include which o f the following? (Mark all that are true.)

○ Recommend a diet rich in calcium and vitamin D ○ Begin c a lcium and vitamin D supplementation ○ Recommend adequate weight‐bearing exercise ○ B e g in a bispho spho n a te ○ B e g in raloxifene (Evista) ○ Repeat the DXA scan in 1 year 43.A n 18‐year‐old female sees you because she is con cerned about heavy menstrual periods. She reports that her periods have a lways been regular, occurring every 2 8 days, but last 8 – 1 0 days and require her to change her pads every 1 – 2 hours. On examination her vital signs are normal and the physical examination is otherwise unremarkable. 44 Of the following underlying etiologies, which one is most likely in this patient?

A) Hyperprola ctinemia B) Polycystic ovary syndrome C)A blee d ing d isord er D)Endo m e trial hyperplasia E) Endometrial cancer 44. A 53‐year‐old female sees you for treatment of hot flashes. She is not interested in hormonal treatment because she has a sister with breast c anc e r. N o nho rmo nal methods that have the best e v idenc e for re lieving this patient’s vasomotor symptoms include which of the following? (Mark all that are true.)

○ Relaxation ○ Exercise ○ Cognitive ‐behavioral therapy ○ Paroxetine (Paxil) ○ Blac k cohosh ○ Acupuncture 45.Interventions shown to decrease the incidence of genital warts include which of the following? (Mark all that are true.)

○ HPV vacc ine (Gardasil 9) ○ C o ndo m use ○ Skin‐to ‐skin genital contact instead of penetrative intercourse 1. A 48-year-old female sees you because of vulvar itching. She has self-treated this problem with over-the- counter antifungal medications multiple times and was subsequently diagnosed with a chronic candidal Question 46 infection and treated with multiple courses of fluconazole (Diflucan). She is not currently sexually active and is not using any medications. On a review of systems she notes intermittent pain with defecation. She has no other health problems.

On examination you note white atrophic papules and a few plaques on the labia majora. There is some fissuring at the posterior fourchette, making insertion of the speculum painful. The vaginal mucosa appears normal, with a thin, white, normal-appearing discharge. There is lichenification and excoriation evident in the perianal region. The remainder of the skin examination is normal. An office wet mount is negative.

Which one of the following is the most likely diagnosis?

A) Vulvar candidiasis

B) Atrophic vaginitis

C) Lichen planus

D) Lichen sclerosus

E) Atopic dermatitis

F) Vitiligo

Lichen sclerosus (LS) is a chronic, progressive dermatologic disorder that is benign. It most commonly involves the anogenital region but can occur on other skin surfaces. It usually manifests as inflammation, epithelial thinning, and dermal changes. Common symptoms include pruritus, pain, dyspareunia, and pain with defecation. The etiology is unknown but it commonly occurs in peri- or postmenopausal women and prepubertal girls. LS is frequently missed or inaccurately diagnosed, leading to a delay in therapy.

Early identification and treatment of LS is important to decrease the likelihood of progressive disease and disfigurement of the vulvar architecture. Women with vulvar LS are also at increased risk for squamous cell carcinoma. It is recommended that even asymptomatic women with LS receive treatment. The foundation of therapy is topical corticosteroids. Intralesional corticosteroids can also be effective for more hypertrophic, thickened plaques.

Candidiasis is also marked by vulvar pruritus but is usually more erythematous and is less likely to cause pain with defecation. LS is sometimes misdiagnosed as candidiasis, especially if the health care provider does not perform a physical examination. Atrophic plaques and papules are also less consistent with vulvar candidiasis. Atrophic vaginitis usually presents with vaginal dryness, burning, irritation, and dyspareunia. The tissue appears thin with a loss of rugae, but plaques and papules would be unusual, as would involvement of the perianal region.

Lichen planus also causes pruritus and loss of genital architecture, but it usually involves the vagina, and lesions are more typically bright and erythematous with white erosions or serpentine borders. A punch biopsy will distinguish lichen planus from LS. The absence of lesions in the vagina in this patient makes lichen planus less likely.

Vulvar dermatitis is also associated with itching and irritation, but erythema and scale are usually more marked. When atopic dermatitis involves the vulva there are usually other signs of dermatitis elsewhere on the skin. Early LS can look very similar to dermatitis and a biopsy is helpful to differentiate the two conditions. Vitiligo causes whitening of the skin but is not characterized by intense pruritus and pain.

Prabhu A, Gardella C: Common vaginal and vulvar disorders. Med Clin North Am 2015;99(3):553-574. O’Connell TX, Nathan LS, Satmary WA, Goldstein AT: Non-neoplastic epithelial disorders of the vulva. Am Fam Physician 2008;77(3):3231-326. 1. A 24-year-old patient who did not know she was pregnant has an ultrasound examination that reveals a missed spontaneous abortion at 8 weeks gestation. Appropriate advice regarding her options includes which of the following? (Mark all that are true.)

○ Expectant management is limited to 2 weeks

○ Misoprostol (Cytotec) or uterine aspiration would be more effective than expectant management for anembryonic gestation or embryonic demise

○ Misoprostol for the treatment of missed spontaneous abortion is effective only when administered vaginally

○ Methotrexate is an appropriate medication for missed spontaneous abortion

Several treatment regimens using oral or vaginal misoprostol (or a combination) have been studied in the setting of spontaneous abortion. A Cochrane review looked at studies regarding expectant versus surgical treatment. Rates of infection, as well as mental health outcomes, were similar, but more women required intervention and blood transfusions in the expectant management group. The conclusion was that a woman’s preference should guide decision making.

There is no time limit for expectant management. Misoprostol is safe and effective for the treatment of missed spontaneous abortion, whereas methotrexate is used for medical termination of pregnancy or ectopic pregnancy and is not indicated for spontaneous abortion/miscarriage.

Prine LW, MacNaughton H: Office management of early pregnancy loss. Am Fam Physician 2011;84(1):75-82.

1. Which one of the following phases of is characterized by high progesterone levels and low FSH and LH levels?

A) The follicular-proliferative phase

B) The follicular-secretory phase

C) The luteal-proliferative phase

D) The luteal-secretory phase

E) The ovulatory phase

Critique: Several treatment regimens using oral or vaginal misoprostol (or a combination) have been studied in the setting of spontaneous abortion. A Cochrane review looked at studies regarding expectant versus surgical treatment. Rates of infection, as well as mental health outcomes, were similar, but more women required intervention and blood transfusions in the expectant management group. The conclusion was that a woman’s preference should guide decision making.

There is no time limit for expectant management. Misoprostol is safe and effective for the treatment of missed spontaneous abortion, whereas methotrexate is used for medical termination of pregnancy or ectopic pregnancy and is not indicated for spontaneous abortion/miscarriage.

Prine LW, MacNaughton H: Office management of early pregnancy loss. Am Fam Physician 2011;84(1):75-82. 48

1. Which one of the following phases of menstruation is characterized by high progesterone levels and low FSH and LH levels?

A) The follicular-proliferative phase

B) The follicular-secretory phase

C) The luteal-proliferative phase

D) The luteal-secretory phase

E) The ovulatory phase

Critique: The normal menstrual cycle can be divided into two segments: the ovarian cycle and the uterine cycle. The ovarian cycle is further divided into the follicular, ovulatory, and luteal phases. Similarly, the endometrium cycles through the menstrual, proliferative, and secretory phases with the secretory phase corresponding to the ovulatory cycle’s luteal phase. At the beginning of the menstrual cycle, during the ovarian , gonadal hormone levels (estrogen and progesterone) and LH levels are low and FSH levels begin to rise. This stimulates the growth of follicles that begin to secrete estrogen,which is the stimulus for the uterine proliferative phase of the endometrium. Just prior to ovulation, an LH surge is produced, accompanied by a smaller increase in FSH levels. After ovulation, there is a transition to the luteal-secretory phase, which is characterized by high estrogen and progesterone levels secreted by the , which in turn act centrally to suppress the pituitary release of gonadotropin hormones, resulting in low levels of FSH and LH.Knudtson J, McLaughlin JE: Female reproductive endocrinology. Merck Manual Professional Version, 2016 49

1. A 35-year-old female comes to your office with cyclic pelvic pain and dyspareunia. Her sister was recently diagnosed with endometriosis that was found incidentally during an appendectomy.

Which one of the following is the gold standard for diagnosing endometriosis?

A) A bimanual pelvic examination

B) A CA-125 test

C) Pelvic ultrasonography

D) Pelvic MRI

E) Diagnostic laparoscopy

Critique: Endometriosis is a condition where endometrial tissue is found outside the uterus. These implants are most commonly found in the pelvis, but case studies have documented endometrial implants throughout the body, including in the pleura and brain. Up to 10% of women in the United States have endometriosis and it is an important cause of pelvic pain, dyspareunia, and infertility.

The initial evaluation of endometriosis is a clinical one in a primary care office, but the gold standard for diagnosing endometriosis is surgical (SOR A). The clinical history of cyclic pain and dyspareunia suggests endometriosis, and most clinicians will begin suppressive therapy (NSAIDs, combined oral contraceptives, and possibly gonadotropin-releasing hormone inhibitors) prior to surgery. There is no imaging modality that reliably detects all possible endometriosis lesions.

Many women undergo ultrasonography in the evaluation of pelvic pain. It sometimes detects an endometrial mass and can exclude other causes. However, negative ultrasonography does not exclude the diagnosis of endometriosis (SOR A). Similarly, pelvic MRI will often detect endometrial lesions, but a negative MRI does not exclude the diagnosis of endometriosis (SOR A).

Bimanual pelvic examination is neither sensitive nor specific for diagnosing endometriosis (SOR C). The CA-125 biomarker is being studied for diagnosing endometriosis but is not sensitive or specific enough to be useful (SOR A).

Schrager S, Falleroni J, Edgoose J: Evaluation and treatment of endometriosis. Am Fam Physician 2013;87(2):107-113. Nisenblat V, Bossuyt PM, Shaikh R, et al: Blood biomarkers for the non-invasive diagnosis of endometriosis. Cochrane Database Syst Rev 2016;(5):CD012179. 1. A healthy 40-year-old female presents for a routine health maintenance visit. She has no health concerns but 50 brings a list of requests for screening, as she has not been to a physician since the birth of her youngest child 8 years ago. Her family history is significant for depression and heart disease. She has never smoked and rarely drinks alcohol. She is divorced and is in a monogamous relationship with a new sex partner. She manages an accounting office. She exercises sporadically but follows a well-balanced diet with 3–4 servings of dairy each 2 day. She has a blood pressure of 137/86 mm Hg and a BMI of 25.8 kg/m . A physical examination is otherwise unremarkable.

U.S. Preventive Services Task Force recommendations for this visit include which of the following? (Mark all that are true.)

○ Counseling regarding physical activity

○ A clinical breast examination

○ A pelvic examination

○ Cervical cancer screening

○ Screening for diabetes mellitus

○ Chlamydia screening

○ Mammography

Critique: The U.S. Preventive Services Task Force (USPSTF) recommends screening for cervical cancer in women 21–65 years of age with a Papanicolaou (Pap) test every 3 years or, for women age 30–65 years who want to lengthen the screening interval, screening with a combination of cytology and HPV testing every 5 years (grade A recommendations).

For patients who are overweight or obese who have additional cardiovascular risk factors, the USPSTF recommends counseling regarding a healthful diet and physical activity for the prevention of cardiovascular disease (grade B recommendation). The USPSTF also recommends screening for abnormal glucose in adults 40–70 years of age who are overweight or obese (B recommendation).

The USPSTF recommends Chlamydia screening in sexually active women age 24 years and younger and in older women who are at increased risk of infection (grade B recommendations). Risk factors for infection include a new sex partner, more than one sex partner, a sex partner with concurrent partners, or a sex partner who has a sexually transmitted infection; inconsistent condom use by persons who are not in mutually monogamous relationships; a previous or coexisting sexually transmitted infection; and exchanging sex for money or drugs.

Mammographic screening should begin at age 50 in low-risk women (USPSTF grade B recommendation). The decision to start regular biennial screening mammography before age 50 should be an individual one and take patient context into account, including the patient’s values regarding specific benefits and harms. The USPSTF has concluded that the current evidence is insufficient to assess the additional benefits and harms of clinical breast examination beyond screening mammography in women 40 years of age or older.

Screening for osteoporosis is not recommended in low-risk women under age 65, and screening pelvic examinations are not recommended.

The Agency for Healthcare Research and Quality has an electronic preventive services selector function available on its website or as a smartphone app. Individual patient information can be entered to receive evidence-based recommendations for health care. This can be accessed at http://epss.ahrq.gov.

Published recommendations. US Preventive Services Task Force, 2016. Agency for Healthcare Research and Quality: Search for recommendations, 2016.

51 1. A 23-year-old female with nausea and dull left adnexal pain presents to your office for evaluation. Her pain began about 48 hours ago. She has no vaginal discharge or urinary or gastrointestinal symptoms. She is monogamous with a male sexual partner and they consistently use condoms during intercourse. Her last menstrual period was about 31 days ago. She was seen in the emergency department last evening. A urine hCG was negative and transvaginal ultrasonography showed a unilocular, thin-walled, anechoic lesion measuring 45 mm on her left ovary. She took 500 mg of naproxen for pain last night before she went to bed. She recalls a similar episode of pain last year but is not sure whether it was on the left or right side. It resolved over a couple of weeks.

Which one of the following would be appropriate in the management of the patient’s ovarian cyst at this point?

A) Repeat ultrasonography in 2 weeks B) Monitoring to see if the cyst will resolve spontaneously

C) Oral contraceptive pills to hasten the resolution of the cyst

D) Immediate laparoscopy to rule out malignancy

Critique: Ovarian cysts are common and physiologically normal in ovulating women of reproductive age. It is important to consider other causes of a painful adnexal mass (infection, malignancy, ectopic pregnancy, ovarian torsion) in appropriate clinical scenarios. Ruling out occult pregnancy is important in a young woman with adnexal pain. Initial ultrasonography is normal in approximately 15%–26% of women with an ectopic pregnancy.

When larger cysts rupture they release blood or cystic fluid that can result in peritoneal irritation and pain. However, expectant management is appropriate because most resolve spontaneously in 8–12 weeks without the need for surgical intervention. Follow-up ultrasonography is recommended in 4–6 weeks. Surgical management is indicated only if the cyst persists beyond 8–12 weeks, or if other high-risk signs are present on ultrasonography.

Combination oral contraceptive pills do inhibit ovulation and prevent cyst formation, but they do not reduce the time it takes for an existing cyst to resolve (SOR B).

Braffman BH, Coleman BG, Ramchandani P, et al: Emergency department screening for ectopic pregnancy: A prospective US study. Radiology 1994;190(3):797-802. Saunders BA, Podzielinski I, Ware RA, et al: Risk of malignancy in sonographically confirmed septated cystic ovarian tumors. Gynecol Oncol 2010;118(3):278-282. Grimes DA, Jones LB, Lopez LM, Schulz KF: Oral contraceptives for functional ovarian cysts. Cochrane Database Syst Rev 2014;(4):CD006134 52

1. A 22-year-old female comes to your office with concerns about lower abdominal pain and vaginal discharge. She has been sexually active with one male partner for the past 2 months and has a lifetime total of three partners. She has no previous history of sexually transmitted infections. On examination she is afebrile and has cervical motion tenderness but no adnexal tenderness or masses.

True statements regarding this patient’s diagnosis and treatment include which of the following? (Mark all that are true.)

○ Transvaginal ultrasonography should be ordered to rule out tubo-ovarian abscess ○ She should be treated presumptively for pelvic inflammatory disease (PID) without waiting for the results of testing for specific infections

○ Antibiotic regimens that include a fluoroquinolone are recommended for treatment ○ Her symptoms are most likely due to Chlamydia or gonorrhea

○ If she has an IUD in place it should be removed as soon as the diagnosis of PID is made Critique: Ultrasonography is not routine in the evaluation of pelvic inflammatory disease (PID) but should be considered in women with severe symptoms, as well as those who don’t improve after 72 hours of treatment. PID can be treated with either parenteral or oral antibiotics. Women with severe nausea, vomiting, pain, fever, pregnancy, or tubo-ovarian abscesses should be hospitalized and should receive intravenous antibiotics. Recommended parenteral treatment includes cefotetan and doxycycline (which can be given orally) or clindamycin and gentamicin. Women who are treated as outpatients can get a combination of intramuscular ceftriaxone and oral doxycycline, intramuscular cefoxitin/probenecid, or a third-generation cephalosporin and oral doxycycline with or without oral metronidazole. Single-dose erythromycin is not appropriate. A woman can receive treatment for PID with an IUD in place.

Less than half of women with PID test positive for gonorrhea or Chlamydia. Other potential causes include anaerobic bacteria, Mycoplasma, Ureaplasma, and Streptococcus. Because Neisseria gonorrhoeae is increasingly resistant to quinolones, they are no longer recommended for treatment of PID.

2015 Sexually transmitted diseases treatment guidelines: Pelvic inflammatory disease (PID). Centers for Disease Control and Prevention, 2015. 53

1. Factors that may influence smoking cessation recommendations for women as compared to men include which of the following? (Mark all that are true.)

○ Women are more susceptible to social cues that trigger smoking ○ Women are less susceptible to sensory triggers such as smell and taste

○ Women are more concerned about postcessation weight gain

○ Withdrawal symptoms tend to be less intense in women

○ Nicotine replacement methods are more effective in reducing cravings

a The 2001 Surgeon General’s report—Women and smoking. US Dept of Health and Human Services, Office on Smoking and Health, 2001. Committee Opinion No. 503: Tobacco use and women’s health. Obstet Gynecol 2011;118(3):746-750. How sex/gender influence health & disease (A-Z). National Institutes of Health, 2016. Research Report Series: Tobacco/nicotine. National Institute on Drug Abuse. NIH Publication No. 16-4342. Reviewed 2012. Modified 2016. 54

1. A 22-year-old college student comes to your office and asks to be tested for sexually transmitted infections. The medical assistant reports that the patient was upset when she saw how much she weighed. On questioning the patient admits that for the past year she has experienced episodes of uncontrollable eating followed by self-induced vomiting. Her weight is normal.

True statements regarding treatment for this condition include which of the following? (Mark all that are true.)

○ Cognitive-behavioral therapy has been shown to improve symptoms

○ SSRIs can be beneficial in patients who do not respond to initial treatment

○ Syncope is an indication for hospitalization

○ More than half of patients will relapse after treatment

Critique: Although up to half of patients with anorexia will relapse after treatment, only 20% of those with bulimia will relapse. Up to 90% of patients with eating disorders are women. The reason for this is multifactorial but it is thought to be mainly due to societal influences. Bulimia can be treated with cognitive-behavioral therapy and with SSRIs if counseling is not effective. Medications have not been proven to help symptoms of anorexia, unless the patient has comorbid depression or anxiety. Criteria for inpatient hospitalization for bulimia include syncope, a serum potassium level <3.2 mEq/L, a serum chloride level <88 mEq/L, esophageal tears, cardiac arrhythmias including a prolonged QTc, hypothermia, suicide risk, intractable vomiting, hematemesis, and failure to respond to outpatient treatment.

Harrington BC, Jimerson M, Haxton C, Jimerson DC: Initial evaluation, diagnosis, and treatment of anorexia nervosa and bulimia nervosa. Am Fam Physician 2015;91(1):46-52. 55 1. You are asked to develop an educational campaign to help women in your community understand the risk of heart disease and to help them identify and use appropriate preventive strategies. Which of the following would be appropriate information to include in a program to educate women about heart disease and its prevention? (Mark all that are true.)

○ Framingham scores are used to assess cardiovascular disease risk in men but have not been validated in women

○ Smoking confers the same risk for coronary artery disease in women as in men ○ Chest pain is uncommon in women with acute myocardial infarction

○ Women are less likely than men to die as a result of their first myocardial infarction ○ The American Heart Association says women need a minimum of 150 minutes per week of moderate- intensity physical activity

○ Hormone therapy after menopause confers cardioprotective benefits in women

Critique: Women and men differ in their risk for and presentation of heart disease. Women also respond differently to symptoms and to preventive strategies and treatment. Further compounding this problem, many health care providers do not appreciate the burden of cardiac disease in women, which can further delay or hinder their care.

Using the Framingham risk calculator can help individualize risk-reduction strategies in women as well as in men. The first step in applying the guidelines to individual women is the assessment of overall cardiovascular disease risk. The Framingham 10-year coronary heart disease risk score is central to making appropriate recommendations for the prevention of coronary heart disease. Risk levels can be calculated using age, sex, total and HDL-cholesterol levels, smoking history, and blood pressure. An online calculator is available at http://www.cvriskcalculator.com. Patients at high risk have a greater than 20% 10-year coronary heart disease risk. Patients at intermediate risk have a 10%–20% 10-year coronary heart disease risk, and patients at lower risk have a less than 10% 10-year coronary heart disease risk. “Optimal risk” is defined as optimal levels of all risk factors and adherence to a heart-healthy lifestyle (SOR A).

Although fewer adult women smoke than men (15% versus 19% of men), a recent meta-analysis reported that in all age groups, with the exception of the youngest (30–44 years), the additional risk for coronary artery disease conferred by cigarette smoking is 25% greater in women than in men. Furthermore, the combination of smoking with oral contraceptive use has a synergistic effect on the risk of acute myocardial infarction, stroke, and venous thromboembolism (SOR A). In addition, fewer women survive their first myocardial infarction compared to men.

Women with heart disease have a higher prevalence of angina, a lower burden of obstructive coronary artery disease on angiography, and a poorer prognosis compared to men. Current risk scores, based on American College of Cardiology/American Heart Association thresholds determined in predominantly male-based populations, may not accurately predict risk in women.

Optimal recognition and timely management of acute myocardial infarction is critical. Although chest pain is the most common presenting symptom in both men and women, women were more likely than men to present without chest pain and have higher mortality than men, especially in younger age groups.

Women should be advised to accumulate at least 150 minutes per week of moderate exercise, 75 minutes per week of vigorous exercise, or an equivalent combination of moderate- and vigorous-intensity aerobic physical activity. Aerobic activity should be performed in at least 10-minute increments, preferably spread throughout the week. Additional cardiovascular benefits are provided by increasing moderate-intensity aerobic physical activity to 300 minutes per week, vigorous-intensity physical activity to 150 minutes per week, or all activity to an equivalent combination of both. Observational data demonstrates an association between higher levels of physical activity and lower rates of many chronic diseases, including cardiovascular disease, as well as enhanced longevity. In addition, higher levels of activity were associated with commensurately lower rates of atherosclerotic cardiovascular disease in a curvilinear fashion (SOR B).

Premenopausal women are relatively protected against cardiovascular disease compared with age-matched men. However, this sex gap narrows after menopause. Estrogen therapy does not confer cardioprotective effects in postmenopausal women.

Mosca L, Benjamin EJ, Berra K, et al: Effectiveness-based guidelines for the prevention of cardiovascular disease in women—2011 update: A guideline from the American Heart Association. Circulation 2011;123(11):1243-1262. Final Recommendation Statement: Aspirin Use to Prevent Cardiovascular Disease and Colorectal Cancer: Preventive Medication. US Preventive Services Task Force, 2016. Hayes SN: Preventing cardiovascular disease in women. Am Fam Physician 2006;74(8):1331-1340. Garcia M, Mulvagh SL, Merz CN, et al: Cardiovascular disease in women: Clinical perspectives. Circ Res 2016;118(8):1273-1293. 56 1. A 29-year-old female comes to your office with symptoms of an upper respiratory infection. During the visit she mentions that she ran out of birth control pills 2 months ago and would like to restart them. The last time she had intercourse was 4 days ago and she did not use condoms. Her last menstrual period was 2 weeks ago. She is overdue for a Papanicolaou test and declines one today. Her vital signs are within the normal range and the physical examination is otherwise unremarkable except for the upper respiratory infection. A urine pregnancy test is negative.

Contraceptive methods that could be safely started today include which of the following? (Mark all that are true.)

○ A combined estrogen/progestin pill

○ A norelgestromin/ethinyl estradiol patch (Ortho Evra)

○ An etonogestrel/ethinyl estradiol vaginal ring (NuvaRing)

○ A copper IUD (ParaGard)

○ A levonorgestrel IUD (Mirena)

○ Medroxyprogesterone acetate (Depo-Provera)

Critique: Delaying contraception can lead to poor adherence. Using a flexible, evidenced-based approach to starting contraception can decrease the rate of unintended pregnancy. and emergency contraception do not accelerate cervical dysplasia or neoplasia, and do not interfere with cervical cytology. A Papanicolaou test is not necessary before initiating any form of birth control.

Combined hormonal contraception such as the combined estrogen/progestin pill, the etonogestrel/ethinyl estradiol vaginal ring, or the norelgestromin/ethinyl estradiol patch can be started at any point in the menstrual cycle. It is still possible that this patient is pregnant, but starting combined hormonal contraception will not harm the fetus. A urine pregnancy test should be checked in 2 weeks, and if it is negative the method can be continued. Continuation rates are enhanced when patients take their first pill immediately after requesting birth control (SOR C).

Insertion of a copper IUD within 5 days of unprotected intercourse is the most effective form of emergency contraception. It is also an excellent choice for ongoing contraception. The insertion of a levonorgestrel IUD is not recommended for this patient today because it is possible that the patient is pregnant and it is too early for β-hCG detection in urine. The recommendation is to use another birth control method (pill, patch, ring) to bridge, and repeat the urine pregnancy test in 2 weeks. If that test is negative the levonorgestrel IUD can be inserted then.

Initiating medroxyprogesterone acetate on any day of a patient’s menstrual cycle is safe and effective and can decrease unintended . This patient should have a repeat pregnancy test in 2 weeks, but medroxyprogesterone acetate will not harm the fetus.

It is recommended that a backup barrier method, such as male condoms, be used for 7 days for women who are not in the first 5 days of their menstrual cycle. If hormonal emergency contraception is administered the new birth control method should not be started until the next day.

Lesnewski R, Prine L: Initiating hormonal contraception. Am Fam Physician 2006;74(1):105-112. Scott T: Same-day initiation of hormonal contraceptives. Am Fam Physician 2009;80(7):690-691. Quick start algorithm. Reproductive Health Access Project, 2016. 57

1. According to the Advisory Committee on Immunization Practices, which of the following previously unimmunized females should receive the HPV vaccine? (Mark all that are true.)

○ A 12-year-old presenting for a well child visit

○ A 16-year-old with a history of multiple sexual partners who just found out she is pregnant ○ A 24-year-old with HIV

○ A 27-year-old with ASCUS on a Papanicolaou smear and a positive test for HPV type 16 ○ A 32-year-old with a new sexual partner Critique: The Advisory Committee on Immunization Practices (ACIP) recommends immunizing females against HPV starting at age 11 or 12 years, but it is possible to start at 9 years of age (SOR C). The ACIP recommends immunizing males as early as age 11 or 12 and up to age 26 (SOR C). Evidence suggests that a two-dose regimen is as effective as three doses if given before age 15.

HPV vaccination thus far appears to be safe and immunogenic in the presence of HIV (SOR C). Administration of the HPV vaccine is not contraindicated in persons 9–26 years of age with HIV infection and is reasonable to consider for this age group, as recommended for the uninfected population.

The FDA has approved the HPV vaccine for males and females under age 26. The 27-year-old female’s age excludes her from this group. If she were 24 it would be appropriate to give her HPV vaccine even though she is positive for HPV type 16, as the vaccine would protect her from infection with other HPV subtypes (SOR C).

HPV vaccine is not approved for use in pregnancy (SOR A), so it is not recommended for the 16-year-old female. Despite the 32-year-old female's increased risk due to a possible new exposure, HPV vaccine is only approved for use up to age 26 (SOR

Hecht FM, Luetkemeyer A: HIV InSite: Immunizations and HIV: HPV vaccination in HIV infection, 2011. Markowitz LE, Dunne EF, Saraiya M, et al: Human papillomavirus vaccination: Recommendations of the Advisory Committee on Immunization Practices (ACIP). MMWR Recomm Rep 2014;63(RR-05):1-30. Petrosky E, Bocchini JA, Hariri S, et al: Use of 9-valent human papillomavirus (HPV) vaccine: Updated HPV vaccination recommendations of the advisory committee on immunization practices. MMWR Morb Mortal Wkly Rep 2015;64(11):300-304. Van Damme P, Meijer CJ, Kieninger D, et al: A phase III clinical study to compare the immunogenicity and safety of the 9-valent and quadrivalent HPV vaccines in men. Vaccine 2016;34(35):4205-4212. Meites E, Kempe A, Markowitz LE: Use of a 2-dose schedule for human papillomavirus vaccination—Updated recommendations of the Advisory Committee on Immunization Practices. MMWR Morb Mortal Wkly Rep 2016;65(49):1405-1408. 58 1. A 22-year-old sees you to establish care after moving to your area. She reports a recent history of new headaches and describes a unilateral visual disturbance that is scintillating and starts before the headache. This disturbance usually lasts about 30 minutes and resolves before the headache pain begins. She has never been pregnant and does not wish to become pregnant in the next few years, and her medications include norgestimate/ethinyl estradiol (Ortho-Cyclen), 0.25 mg/35 µg. She was diagnosed with pelvic inflammatory disease 5 months ago and completed treatment. She is a nonsmoker and her vital signs are normal today.

According to the CDC’s 2016 Medical Eligibility Criteria for Contraception Use, contraceptive options considered to be safe for this patient include which of the following? (Mark all that are true.)

○ lower dose of a combined estrogen/progestin pill (25 µg ethinyl estradiol)

○ Medroxyprogesterone acetate (Depo-Provera)

○ The etonogestrel/ethinyl estradiol vaginal ring (NuvaRing)

○ A copper IUD (ParaGard)

○ A levonorgestrel IUD (Mirena)

○ An etonogestrel implant (Nexplanon)

Critique: The CDC and the World Health Organization provide guidelines regarding considerations and contraindications for contraception in women, using the following classification system:

Category 1: A condition for which there is no restriction for the use of the contraception method Category 2: A condition for which the advantages of using the method generally outweigh the theoretical or proven risks Category 3: A condition for which the theoretical or proven risks usually outweigh the advantages of using the method Category 4: A condition that represents an unacceptable health risk if the contraceptive method is used

This patient has migraine with aura. Aura is a specific focal neurologic symptom that is classically identified by three findings: the onset of a visual disturbance before the headache; a visual disturbance that lasts less than an hour, usually between 15 and 45 minutes; and resolution of the visual disturbance before the pain of the headache begins. The history also often reveals a sign, in that women describing aura often raise a single hand to the side of the face or head to demonstrate the location of the aura.

Migraine with aura is a category 4 condition for estrogen-containing contraception. This patient’s current combined hormone pills should be stopped and she should be switched to a non–estrogen-containing contraceptive. It is not appropriate to switch her to a combined pill with a lower dose of estrogen or to initiate the etonogestrel/ethinyl estradiol vaginal ring or the norelgestromin/ethinyl estradiol patch.

Migraine with aura is category 1 for initiation of the levonorgestrel IUD or copper IUD with aura. This patient’s nulliparous status is category 2, and her history of pelvic inflammatory disease more than 3 months ago is also category 2. Migraine with aura is category 1 for initiation of the etonogestrel implant, as well as for initiation of medroxyprogesterone acetate

Headache Classification Subcommittee of the International Headache Society: The International Classification of Headache Disorders: 2nd edition. Cephalalgia 2004;24(Suppl 1):9-160. Curtis KM, Tepper NK, Jatlaoui TC, et al: US Medical Eligibility Criteria for Contraceptive Use, 2016. MMWR Recomm Rep 2016;65(3):1-103. 59

1. Physiologic factors that contribute to sex-based differences in drug activity include which of the following characteristics in women compared to men? (Mark all that are true.)

○ Slower gastric emptying time

○ A lower glomerular filtration rate

○ A larger relative volume of adipose tissue

○ Increased gastric acid secretion Critique: A number of factors influence medication absorption, metabolism, and excretion in women, and women are 50%– 75% more likely to experience an adverse drug reaction than men. Compared to men, women have lower body weight, slower gastrointestinal motility, less intestinal enzymatic activity, and a lower glomerular filtration rate. Slower gastric emptying times may mean that medications that should be taken on an empty stomach, such as levothyroxine, require a longer interval after a meal (SOR C). Women have a larger relative volume of adipose tissue, affecting distribution of lipophilic medications such as benzodiazepines (SOR C). Women also have a lower glomerular filtration rate, resulting in slower excretion of renally cleared medications such as digoxin (SOR C). Women secrete less gastric acid, leading to decreased absorption of certain medications such as ketoconazole (SOR C).

Whitley H, Lindsey W: Sex-based differences in drug activity. Am Fam Physician 2009;80(11):1254-1258. 1. A 66-year-old female sees you to establish care after moving from out of state. She brings a copy of her medical records and you see she has a history of cervical intraepithelial neoplasia (CIN) 3 removed by loop electrosurgical excision procedure (LEEP) 2 years ago with clear surgical margins. She had co- testing at age 65 that showed normal cytology and was negative for high-risk HPV. She is otherwise in good health and uses no medications.

Which one of the following should you recommend for additional Papanicolaou screening?

A) No additional screening

B) Co-testing now, and if normal, repeated in 3 years then every 5 years until age 84

C) Co-testing one time, in 4 years

D) Co-testing in 4 years, then once more 5 years later

E) Referral for colposcopy with endocervical sampling

Massad LS, Einstein MH, Huh WK, et al: 2012 updated consensus guidelines for the management of abnormal cervical cancer screening tests and cancer precursors. J Low GenitTract Dis 2013;17(5 Suppl 1):S1-S27.